OU blog

Personal Blogs

Valentin Fadeev

Self ex-spline-atory

Visible to anyone in the world
Edited by Valentin Fadeev, Thursday, 27 Mar 2014, 10:06

M832 did not really fit in the "big picture" of my study plan this year, not only because I am notoriously bad at numerical calculations. Having invested a lot of effort and sleepless nights in developing intuition for the behaviour of analytic functions I was suddenly confronted by the cubic splines which seemed to have all those properties, that well-mannered functions would be never allowed to possess.

Nicely, but artificially glued together of several pieces of cubics, smooth only up to the second derivative, vanishing on the entire intervals, now this is what seems really counter-intuitive. Be that as it may, the TMA deadlines had to be met.

The following is the kind of problem I got stuck with for a while. Obviously I am not replicating a TMA question here, giving an extended solution to one of the problems from the Course Notes. So the task is to express a function, say f of x equals x squared in terms of cubic B-splines on the entire real axis. I am omitting a lot of background material focusing on one particular idea that arises in the solution.

equation left hand side x squared equals right hand side n ary summation from p equals negative normal infinity to normal infinity over lamda sub p times cap b sub p of x

Since cap b sub p cubed of x has a supporting interval equation left hand side open p comma sum with, 3 , summands p plus three plus one close equals right hand side open p comma p plus four close of length 4 outside which it vanishes, we can start by expressing the function in terms of cap b sub negative three comma cap b sub negative two comma cap b sub negative one comma cap b sub zero on open 0.1 close and then try to extend the result. Calculating the expressions for the splines on open zero comma one close :

cap b sub negative three of x equals negative one divided by 24 times open x minus one close cubed

cap b sub negative two of x equals one divided by 24 times open three times x cubed minus six times x squared plus four close

cap b sub negative one of x equals one divided by 24 times open sum with, 4 , summands negative three times x cubed plus three times x squared plus three times x plus one close

cap b sub zero of x equals one divided by 24 times x cubed

multiplying by the respective coefficients, summing and equating powers of x on each side we arrive at the following system of equations:

case statement case 1column 1 plus plus minus minus plus plus minus minus lamda minus minus three times times three lamda minus minus two times times three lamda minus minus one lamda zero equals equals zero case 2column 1 plus plus minus minus times times three lamda minus minus three times times six lamda minus minus two times times three lamda minus minus one equals equals zero case 3column 1 plus plus minus minus times times three lamda minus minus three times times three lamda minus minus one equals equals 24 case 4column 1 plus plus plus lamda minus minus three times times four lamda minus minus two lamda minus minus one equals equals zero

Having the solution (guaranteed by the Schoenberg-Whitney theorem): equation left hand side open lamda sub negative three comma lamda sub negative two comma lamda sub negative one comma lamda sub zero close equals right hand side open eight divided by three comma negative four divided by three comma eight divided by three comma 44 divided by three close

Now we want to find all coefficients on each of the intervals open xi sub p comma xi sub p plus four close for the points equation left hand side xi sub i equals right hand side i times h semicolon i equals prefix plus minus of one comma prefix plus minus of two full stop full stop full stop full stop . From the general expression for the B-spline it can be deduced that

cap b sub p of xi sub p plus one equals one divided by 24 times h

cap b sub p of xi sub p plus two equals one divided by six times h

cap b sub p of xi sub p plus three equals one divided by 24 times h

which for h equals one leads to the following recurrence relation:

equation left hand side sum with, 3 , summands lamda sub j minus one plus four times lamda sub j minus two plus lamda sub j minus three equals right hand side 24 times j squared

or, after changing the index

sum with, 3 , summands lamda sub j plus four times lamda sub j plus one plus lamda sub j plus two equals 24 left parenthesis j plus three times right parenthesis squared

Now here is the trick that I came up with and that was not (at least not explicitly) described in the Course Notes or the set book. The last expression can be thought of as a "second-order linear inhomogeneous recurrence relation". The advantage of this approach is that the structure of the solution instantly becomes clear.

The general solution of the corresponding homogeneous relation

sum with, 3 , summands lamda sub j plus four times lamda sub j plus one plus lamda sub j plus two equals zero

is derived in the course notes, using the standard method of solving this type of recurrencies and is given by the following expression:

equation left hand side lamda sub j super h equals right hand side alpha times open negative two minus Square root of three close super j plus beta times open negative two plus Square root of three close super j

It can also be found using generating functions. Not surprisingly it depends on 2 arbitary constants, as it takes 2 initial terms, lamda sub zero and lamda sub negative one to reconstruct the whole sequence from the three-term recurrency. Applying the general ideas from the linear systems we deduce that in order to obtain the general solution of the inhomogeneous recurrency we have to add a particular solution to the expression above.

Since the RHS is the quadratic polynomial it makes sence to look for the particular solution in the form:

equation left hand side lamda sub j super p equals right hand side sum with, 3 , summands a times j squared plus b times j plus c

Substituting this into the original recurrency and gatherig together the powers of j we obtain:

equation left hand side sum with, 5 , summands six times a times j squared plus open 12 times a plus six times b close times j plus eight times a plus six times b plus six times c equals right hand side sum with, 3 , summands 24 times j squared plus 144 times j plus 216

which after equating powers gives the solution equation left hand side open a comma b comma c close equals right hand side open four comma 16 comma 44 divided by three close

Thus the general solution of the inhomogeneous equation is given by the following formula:

equation left hand side lamda sub j equals right hand side sum with, 5 , summands alpha times open negative two minus Square root of three close super j plus beta times open negative two plus Square root of three close super j plus four times j squared plus 16 times j plus 44 divided by three

Now we can use the values of lamda sub zero and lamda sub negative one to determine the constants (bearing in mind that equation left hand side open negative two minus Square root of three close times open negative two plus Square root of three close equals right hand side negative one ):

equation left hand side 44 divided by three equals right hand side sum with, 3 , summands alpha plus beta plus 44 divided by three

equation left hand side eight divided by three equals right hand side negative alpha times open negative two plus Square root of three close minus beta times open negative two minus Square root of three close plus eight divided by three

which gives equation sequence alpha equals beta equals zero . Thus finally:

equation left hand side lamda sub j equals right hand side sum with, 3 , summands four times j squared plus 16 times j plus 44 divided by three

which is the solution of the original problem.

Permalink Add your comment
Share post
Valentin Fadeev

Around the infinity

Visible to anyone in the world
Edited by Valentin Fadeev, Sunday, 18 Sep 2011, 23:23

Now I got really fascinated with this topic. The most exciting part of evaluating integrals using residues is constructing the right contour. There are general guidelines for certain types of integrals, but in most cases the contour has to be tailored for a particular problem. Here is another example. Evaluate the following integral:

cap j equals integral over z sub one under z sub two Square root of sum with, 3 , summands cap a plus two times cap b divided by z plus cap c divided by z squared d z

Where cap a comma cap b comma cap c comma z sub one comma z sub two are real, cap a less than zero comma zero less than z sub one less than z sub two semicolon z sub one comma z sub two
are roots of the integrand.

The approach used in the previous example does not work as the integral along a large circle centered at the origin does not tend to 0. This, in fact, is a clue to the solution, as it prompts to have a look what indeed is happening to the function for large z . But first consider the integrand in the neighborhood of the origin (simple pole):

equation sequence Square root of sum with, 3 , summands cap a plus two times cap b divided by z plus cap c divided by z squared equals one divided by z times Square root of cap a times open z sub one minus z close times open z sub two minus z close equals one divided by z times Square root of cap a times z sub one times z sub two times open one minus z divided by z sub one close super one divided by two times open one minus z divided by z sub two close super one divided by two equals
equation sequence equals one divided by z times Square root of cap a times z sub one times z sub two times open one minus z divided by two times z sub one plus cap o of z squared close times open one minus z divided by two times z sub two plus cap o of z squared close equals one divided by z times Square root of cap a times z sub one times z sub two times open one minus z sub one plus z sub two divided by two times z sub one times z sub two times z plus cap o of z squared close
equation sequence equals one divided by z times Square root of cap a times cap c divided by cap a times open sum with, 3 , summands one plus cap b times cap a divided by cap a times cap c times z plus cap o of z squared close equals one divided by z times Square root of cap c times open sum with, 3 , summands one plus cap b divided by cap c times z plus cap o of z squared close
cap r sub zero of Square root of sum with, 3 , summands cap a plus two times cap b divided by z plus cap c divided by z squared equals Square root of cap c

Now the integrand is regular for large absolute value of z hence it can be expanded in the Laurent series convergent for cap r less than absolute value of z less than normal infinity where cap r is large:
equation sequence Square root of sum with, 3 , summands cap a plus two times cap b divided by z plus cap c divided by z squared equals Square root of cap a times open z sub one minus z close times open z sub two minus z close divided by z squared equals Square root of cap a times open one minus z sub one divided by z close super one divided by two times open one minus z sub two divided by z close super one divided by two equals

equation sequence equals Square root of cap a times open one minus z sub one divided by two times z plus cap o of one divided by z squared close times open one minus z sub two divided by two times z plus cap o of one divided by z squared close equals Square root of cap a times open one minus z sub one plus z sub two divided by two times one divided by z plus cap o of one divided by z squared close equals
equation left hand side equals right hand side Square root of cap a times open sum with, 3 , summands one plus cap b divided by cap a times one divided by z plus cap o of one divided by z squared close

The residue at infinity is defined to be the coefficient at one divided by z with the opposite sign:

cap r sub normal infinity of Square root of sum with, 3 , summands cap a plus two times cap b divided by z plus cap c divided by z squared equals negative cap b divided by Square root of cap a

Now we construct contour C by cutting the real axis along the segment open z sub one comma z sub two close and integrating along the upper edge of the cut in the negative direction, then along a small circle equation left hand side cap c sub z sub one equals right hand side open z colon absolute value of z minus z sub one equals epsilon close and along the lower edge of the cut. As the result of that one of the factors of the integrand increases its argument by equation left hand side e super two times pi times i divided by two equals right hand side e super pi times i
. Finally integrate along a small circle equation left hand side cap c sub z sub two equals right hand side open z colon absolute value of z minus z sub two equals epsilon close

cap j equals sum with, 4 , summands integral over z sub two minus epsilon under z sub one plus epsilon f of z d z plus integral over cap c sub z sub one f of z d z plus e super pi times i times integral over z sub one plus epsilon under z sub two minus epsilon f of z d z plus integral over cap c sub z sub two f of z d z
equation sequence lim over epsilon right arrow zero of integral over cap c sub z sub one f of z d z equals zero times lim over epsilon right arrow zero of integral over cap c sub z sub two f of z d z equals zero
lim over epsilon right arrow zero of cap j equals negative two times integral over z sub one under z sub two f of z d z

Integrating along this contour amounts to integrating in the positive direction along a very large circle centered at infinity. Hence the outside of C is the inside of this large circle which therefore includes both the residue at the origin and at infinity. Therefore, by the residue theorem:

cap j equals two times pi times i times open cap r sub zero plus cap r sub normal infinity close

equation sequence integral over z sub one under z sub two Square root of sum with, 3 , summands cap a plus two times cap b divided by z plus cap c divided by z squared d z equals negative pi times i times open Square root of cap c minus cap b divided by Square root of cap a close equals pi times i times open cap b divided by Square root of cap a minus Square root of cap c close

Permalink Add your comment
Share post
Valentin Fadeev

How comple^x can you get? Continued

Visible to anyone in the world
Edited by Valentin Fadeev, Thursday, 27 Mar 2014, 10:11

As a follow up thought I realized just how much easier it would have been to calculate the residue by definition, i.e. expanding the integrand in the Laurent series to get the coefficient a sub negative one . Let x equals negative one plus xi where xi is small:

equation sequence root of order four over four divided by left parenthesis one plus x times right parenthesis cubed equals root of order four over four divided by xi cubed equals two times e super pi times i divided by four divided by root of order four over four times left parenthesis one minus xi times right parenthesis super one divided by four left parenthesis one minus xi divided by two times right parenthesis super three divided by four divided by xi cubed

Of course, care is needed when choosing the value of the root. It depends on the value of the argument set on the upper bound of the cut. Since I chose it to be 0, the correct value of the root is e super pi times i divided by four .

Now

left parenthesis equation left hand side one minus xi times right parenthesis super one divided by four equals right hand side one minus one divided by four times xi minus three divided by 32 times xi squared plus cap o of xi cubed

left parenthesis equation left hand side one minus xi divided by two times right parenthesis super three divided by four equals right hand side one minus three divided by eight times xi minus three divided by 128 times xi squared plus cap o of xi cubed

left parenthesis one minus xi times right parenthesis super one divided by four left parenthesis equation left hand side one minus xi divided by two times right parenthesis super three divided by four equals right hand side one minus five divided by eight times xi minus three divided by 128 times xi squared plus cap o of xi cubed

Therefore, near x equals negative one the integrand has the following expansion:

equation left hand side root of order four over four divided by left parenthesis one plus x times right parenthesis cubed equals right hand side two times e super pi times i divided by four divided by root of order four over four times open one divided by left parenthesis x plus one times right parenthesis cubed minus five divided by eight left parenthesis x plus one times right parenthesis squared minus three divided by 128 times open x plus one close plus g of x close

where g of x is the regular part of the expansion which is of no interest in this problem.

Hence by definition:

equation sequence cap r sub negative one times f of x equals two times e super pi times i divided by four divided by root of order four over four times open negative three divided by 128 close equals negative three times e super pi times i divided by four divided by 64 times root of order four over four

Permalink
Share post
Valentin Fadeev

How comple^x can you get?

Visible to anyone in the world
Edited by Valentin Fadeev, Thursday, 21 Apr 2011, 23:58

I found this example in a textbook dated 1937 which I use as supplementary material for M828. It gave me some hard time but finally sorted out many fine tricks of contour integration. Some inspiration was provided by the discussion of the Pochhammer's extension of the Eulerian integral of the first kind by Whittaker and Watson.

Evaluate the following integral:

integral over zero under one root of order four over four divided by open one plus x close cubed d x

Consider the integral in the complex plane:

cap j equals contour integral over cap c root of order four over four divided by open one plus z close cubed d z

where contour C is constructed as follows. Make a cut along the segment open zero comma one close of the real axis. Let arg of z equals zero on the upper edge of the cut. Integrate along the upper edge of the cut in the positive direction. Follow around the point z equals one along a small semi-circle equation left hand side cap c sub gamma super one equals right hand side open z colon absolute value of one minus z equals gamma comma normal black letter cap i times z greater than zero close in the clockwise direction. The argument of the second factor in the numerator will decrease by three times pi divided by four . Then proceed along the real axis till z equals cap r and further round the circle equation left hand side cap c sub cap r equals right hand side open z colon absolute value of z equals cap r close in the counter-clockwise direction.

This circle will enclose both branch point of the integrand z equals zero and z equals one , however since the exponents add up to unity the function will return to its initial value:

equation sequence z super one divided by four times e super two times pi times i times one divided by four times open one minus z close super three divided by four times e super two times pi times i times three divided by four equals z super one divided by four times open one minus z close super three divided by four times e super two times pi times i equals z super one divided by four times open one minus z close super three divided by four

This is the reason why we only need to make the cut along the segment open zero comma one close and not along the entire real positive semi-axis. Then integrate from z equals cap r in the negative direction, then along the second small semi-circle equation left hand side cap c sub gamma squared equals right hand side open z colon absolute value of one minus z equals gamma comma normal black letter cap i times z less than zero close around the point z equals one where the argument of the second factor will again decrease by three times pi divided by four . Finally integrate along the lower edge of the cut and along a small circle equation left hand side cap c sub delta equals right hand side open z colon absolute value of z equals delta close around the origin where the argument of the first factor will decrease by equation left hand side two times pi times one divided by four equals right hand side pi divided by two .

contour

As the result of this construction the integral is split into the following parts:

multiline equation row 1 cap j equals sum with, 4 , summands integral over delta under one minus gamma f of z d z plus integral over cap c sub gamma super one f of z d z plus e super negative three times pi times i divided by four times integral over one plus gamma under cap r f of z d z plus integral over cap c sub cap r f of z d z plus row 2 sum with, 4 , summands prefix plus of e super five times pi times i divided by four times integral over cap r under one plus gamma f of z d z plus integral over cap c sub gamma squared f of z d z plus e super pi times i divided by two times integral over one minus gamma under delta f of z d z plus integral over cap c sub delta f of z d z

Two integrals around the small circles add up to an integral over a circle:

equation sequence integral over cap c sub gamma super one f of z d z plus integral over cap c sub gamma squared f of z d z equals integral over cap c sub gamma f of z d z equals integral over zero under two times pi root of order four over four divided by open two minus gamma times e super i times theta close cubed times i times gamma times e super i times theta d theta

lim over gamma right arrow zero of integral over cap c sub gamma f of z d z equals zero

Similarly, the integral around a small circle around the origin vanishes:

equation left hand side integral over cap c sub delta f of z d z equals right hand side integral over zero under two times pi root of order four over four divided by open one plus delta times e super i times theta close cubed times i times delta times e super i times theta d theta

lim over delta right arrow zero of integral over cap c sub delta f of z d z equals zero

Integrals along the segment open one plus gamma comma cap r close cancel out:

multiline equation row 1 equation left hand side e super negative three times pi times i divided by four times integral over one plus gamma under cap r f of z d z plus e super five times pi times i divided by four times integral over cap r under one plus gamma f of z d z equals right hand side e super negative three times pi times i divided by four times integral over cap r under one plus gamma f of z d z plus e super two times pi minus three times pi times i divided by four times integral over cap r under one plus gamma f of z d z equals row 2 equation sequence equals e super negative three times pi times i divided by four times integral over cap r under one plus gamma f of z d z minus e super negative three times pi times i divided by four times integral over cap r under one plus gamma f of z d z equals zero

The integral over the large circle also tends to 0 as R increases. This can be shown using the Jordan's lemma, or by direct calculation:

equation sequence integral over cap c sub cap r f of z d z equals integral over zero under two times pi root of order four over four divided by open one plus cap r times e super theta close cubed times i times cap r times e super i times theta d theta equals integral over zero under two times pi root of order four over four divided by open one divided by cap r plus e super i times theta close cubed times i times e super i times theta divided by cap r d theta

lim over cap r right arrow normal infinity of integral over cap c sub cap r f of z d z equals zero

Finally the only two terms that survive allow us to express the contour integral in terms of the integral along the segment of the real axis:

multiline equation row 1 equation sequence cap j equals integral over zero under one f of z d z minus e super pi times i divided by two times integral over zero under one f of z d z equals open one minus e super pi times i divided by two close times integral over zero under one f of z d z equals e super pi times i divided by four times open e super negative pi times i divided by four minus e super pi times i divided by four close times integral over zero under one f of z d z equals row 2 equation left hand side equals right hand side negative two times i times e super pi times i divided by four times sine of pi divided by four times integral over zero under one f of z d z

The contour cap c encloses the only singularity of the integrand which is the pole of the third order at z equals negative one . Hence, by the residue theorem:

cap j equals two times pi times i times cap r sub negative one times f of z

equation left hand side integral over zero under one f of z d z equals right hand side negative Square root of two times pi times e super negative pi times i divided by four times cap r sub negative one times f of z

The residue can be calculated using the standard formula:

equation sequence cap r sub negative one times f of z equals one divided by two times lim over z right arrow negative one of open open one plus z close cubed times root of order four over four divided by open one plus z close cubed close super double prime equals one divided by two times lim over z right arrow negative one of open root of order four over four close super double prime

Calculation of the derivative can be facilitated by taking logarithm first:

g equals root of order four over four

natural log of g equals one divided by four times natural log of z plus three divided by four times natural log of one minus z

equation sequence g super prime divided by g equals one divided by four times z minus three divided by four times open one minus z close equals one minus four times z divided by four times z times open one minus z close

equation left hand side g super prime equals right hand side one minus four times z divided by four times z super three divided by four times open one minus z close super one divided by four

natural log of g super prime equals natural log of one divided by four plus natural log of one minus four times z minus three divided by four times natural log of z minus one divided by four times natural log of one minus z

equation sequence g super double prime divided by g super prime equals negative four divided by one minus four times z minus three divided by four times z plus one divided by four times open one minus z close equals negative three divided by four times open one minus four times z close times z times open one minus z close

equation left hand side g super double prime equals right hand side negative three divided by 16 times z super seven divided by four times open one minus z close super five divided by four

equation sequence cap r sub negative one times f of z equals one divided by two times lim over z right arrow negative one of negative three divided by 16 times z squared times open one minus z close times z super one divided by four divided by open one minus z close super one divided by four equals negative three times e super pi times i divided by four divided by 64 times root of order four over four

equation sequence integral over zero under one root of order four over four divided by open one plus x close cubed d x equals negative Square root of two times pi times e super negative pi times i divided by four of negative three times e super pi times i divided by four divided by 64 times root of order four over four equals three times pi times root of order four over four divided by 64

Permalink
Share post
Valentin Fadeev

Going beyond dx

Visible to anyone in the world
Edited by Valentin Fadeev, Sunday, 18 Sep 2011, 23:23

This is quite a minor trick and like many things listed here may seem quite trivial. However, this is one of those few occasions when I had the tool in mind, before I actually got the example touse it on. Consider:

equation left hand side d times y divided by d times x equals right hand side one plus two divided by x plus y

which does not really require a great effort to solve. But forget all the standard ways for a moment and add d times x divided by d times x equals one to both parts:

equation left hand side d times open x plus y close divided by d times x equals right hand side two times one plus open x plus y close divided by x plus y

equation left hand side sum with, 3 , summands negative one plus one plus open x plus y close times d times open x plus y close divided by one plus open x plus y close equals right hand side two times d times x

equation left hand side d times open sum with, 3 , summands one plus x plus y close divided by one plus open x plus y close equals right hand side d times open y minus x close

natural log of absolute value of sum with, 3 , summands one plus x plus y equals open y minus x close plus natural log of cap c

equation left hand side sum with, 3 , summands one plus x plus y equals right hand side cap c times exp of y minus x

Hope this can be stretched to use in more complicated cases

Permalink Add your comment
Share post
Valentin Fadeev

Vector horror

Visible to anyone in the world
Edited by Valentin Fadeev, Sunday, 23 Jan 2011, 21:49

Had to do some revision of vector calculus/analysis before embarking on M828.

One point which I was not really missing, but did not quite get to grips with was the double vector product. I remembered the formula:

equation left hand side a right arrow multiplication open b right arrow multiplication c right arrow close equals right hand side b right arrow times open a right arrow dot operator c right arrow close minus c right arrow times open a right arrow dot operator b right arrow close ,

but nevertheless had difficulties applying it in excercises.

The reason whas that that the proof I saw used the expression of vector product in coordinates and comparison of both sides of the equation. However, I was aware of another, purely "vector" argument with no reference to any coordinate system.

Eventually I was able to reproduce only part of it, consulting one old textbook for some special trick. So here's how it goes.

For b right arrow multiplication c right arrow is perpendicular to the plane of b right arrow and c right arrow , a right arrow multiplication open b right arrow multiplication c right arrow close must lie in this plane, therefore:

equation left hand side a right arrow multiplication open b right arrow multiplication c right arrow close equals right hand side lamda times b right arrow plus mu times c right arrow

Dot-multiply both parts by a right arrow :

equation left hand side open a right arrow multiplication open b right arrow multiplication c right arrow close dot operator a right arrow close equals right hand side lamda times open b right arrow dot operator a right arrow close plus mu times open c right arrow dot operator a right arrow close

Since a right arrow multiplication open b right arrow multiplication c right arrow close up tack a right arrow , left-hand side is 0, so:

lamda times open b right arrow dot operator a right arrow close plus mu times open c right arrow dot operator a right arrow close equals zero reverse solidus qquod open asterisk operator close

Now define vector c prime right arrow lying in the plane of b right arrow and c right arrow , perpendicular to c right arrow and directed so that c prime right arrow , c right arrow and b right arrow multiplication c right arrow form the left-hand oriented system. This guarantees that the angle between b right arrow and c prime right arrow , open times times b right arrow c prime right arrow hat close less than pi divided by two .

Dot-multiply both parts by c prime right arrow :

equation left hand side open a right arrow multiplication open b right arrow multiplication c right arrow close dot operator c prime right arrow close equals right hand side lamda times open b right arrow dot operator c prime right arrow close

However, equation left hand side open a right arrow multiplication open b right arrow multiplication c right arrow close dot operator c prime right arrow close equals right hand side open c prime right arrow multiplication open b right arrow multiplication c right arrow close dot operator a right arrow close

equation sequence absolute value of b right arrow multiplication c right arrow equals absolute value of b right arrow times absolute value of c right arrow times sine of times times b right arrow c right arrow hat equals absolute value of b right arrow times absolute value of c right arrow times cosine of times times b right arrow c prime right arrow hat

MathJax failure: TeX parse error: Missing or unrecognized delimiter for \right therefore

sine of multiplication multiplication times times c prime right arrow b right arrow c right arrow hat equals one

and

equation sequence absolute value of c prime right arrow multiplication open b right arrow multiplication c right arrow close equals absolute value of c prime right arrow times absolute value of b right arrow times absolute value of c right arrow times cosine of times times b right arrow c prime right arrow hat equals absolute value of c right arrow times open b right arrow dot operator c prime right arrow close

Hence equation left hand side c prime right arrow multiplication open b right arrow multiplication c right arrow close equals right hand side c right arrow times open b right arrow dot operator c prime right arrow close

equation left hand side open a right arrow multiplication open b right arrow multiplication c right arrow close dot operator c prime right arrow close equals right hand side open b right arrow dot operator c prime right arrow close times open a right arrow dot operator c right arrow close

equation left hand side open b right arrow dot operator c prime right arrow close times open a right arrow dot operator c right arrow close equals right hand side lamda times open b right arrow dot operator c prime right arrow close

lamda equals open a right arrow dot operator c right arrow close

mu can be calculated in a similar manner, however, it is easier achieved using equation open asterisk operator close .

Permalink
Share post
Valentin Fadeev

Back to school

Visible to anyone in the world
Edited by Valentin Fadeev, Sunday, 18 Sep 2011, 23:24

This is problem 1.18 from JS (M821 course book). The question is to investigate the motion of a bead that slides on a smooth parabolic wire rotating with constant angular velocity omega about a vertical axis. x is the distance from the axis of rotation.

To simplyfy the calculations we can choose the scale so that equation of the parabola is y equals x squared divided by two .

Yes, I know as a grown up man I should write out the expression of the kinetic energy:

equation sequence cap t equals m times v squared divided by two plus m times omega squared times x squared divided by two equals m divided by two times open d times s divided by d times t close squared plus m times omega squared times x squared divided by two

where v is the tangential velocity of the bead directed along the wire. Then define potential energy as usual:

equation sequence cap v equals negative integral f of y d y equals integral m times g d y equals m times g times y equals m times g times x squared divided by two

bearing in mind that the force of gravity acts in the direction opposite to that of cap y axis, hence changing the sign.

Construct the Lagrangian:

equation sequence script cap l equals cap t minus cap v equals m divided by two times open d times s divided by d times t close squared plus m times omega squared times x squared divided by two minus m times g times x squared divided by two

Define the action:

cap s equals integral script cap l d t

Use variational principle

delta times cap s equals zero

to obtain the Euler-Lagrange equation:

d divided by d times t times open normal partial differential times script cap l divided by normal partial differential times x dot above close minus normal partial differential times script cap l divided by normal partial differential times x equals zero

Calculating separate terms:

equation sequence open d times s divided by d times t close squared equals x dot above squared plus y dot above squared equals x dot above squared times open one plus x squared close

equation left hand side normal partial differential times script cap l divided by normal partial differential times x dot above equals right hand side two times x dot above times open one plus x squared close

equation left hand side d divided by d times t times open normal partial differential times script cap l divided by normal partial differential times x dot above close equals right hand side two times x double dot times open one plus x squared close plus four times x times x dot above squared

equation left hand side normal partial differential times script cap l divided by normal partial differential times x equals right hand side two times x squared times x dot above squared plus two times omega squared times x minus two times g times x

Finally obtain the equation of motion:

x double dot times open one plus x squared close plus open g minus omega squared plus x dot above squared close times x equals zero

However, I am tempted to use an alternative method that I learned on my secondary school physics lessons. It is based on the direct application of Newtons laws and projecting vector equation equation left hand side m times a right arrow equals right hand side n ary summation over cap f right arrow on coordinate axes. Writing out the second law we have to bear in mind that the bead is acted upon by gravitational force and the force of normal reaction which arises due to Newton's third law and acts along the normal to the wire.

graph

Hence Newton's second law is expressed as follows:

equation left hand side m times a right arrow equals right hand side m times g right arrow plus cap n right arrow

Acceleration is split into the tangential and centripetal parts:

equation left hand side a right arrow equals right hand side a right arrow sub tau plus a right arrow sub c

Projecting the equation on the vertcal axis we obtain:

equation left hand side m times a sub tau times sine of alpha plus m times g equals right hand side cap n times cosine of alpha

where alpha is the angle at which the tangent crosses the horizontal axis (hence tangent of alpha equals y dot above divided by x dot above )

Then project on the horizontal axis:

equation left hand side m times a sub c equals right hand side cap n times sine of alpha plus m times a sub tau times cosine of alpha

Eliminate cap n :

cap n equals m divided by sine of alpha times open a sub c minus a sub tau times cosine of alpha close

a sub tau times open sine of alpha plus cosine squared of alpha divided by sine of alpha close minus a sub c divided by tangent of alpha plus m times g equals zero

a sub tau divided by sine of alpha minus a sub c divided by tangent of alpha plus m times g equals zero

Now calculate the tangential acceleration:

equation sequence a sub tau equals d divided by d times t times open d times s divided by d times t close equals d divided by d times t times open Square root of x dot above squared plus y dot above squared close equals d divided by d times t times open x dot above times Square root of one plus x squared close equals x double dot times open one plus x squared close plus x times x two dot above divided by Square root of one plus x squared

and the centripetal part:

equation sequence a sub c equals omega squared times cap r equals omega squared times x

equation sequence sine of alpha equals one divided by Square root of one plus one divided by open y dot above divided by x dot above close squared equals y dot above divided by Square root of x dot above squared plus y dot above squared equals x times x dot above divided by x dot above times Square root of one plus x squared equals x divided by Square root of one plus x squared

equation sequence tangent of alpha equals y dot above divided by x dot above equals x

Plug the above results into the equation:

x double dot times open one plus x squared close plus x times x two dot above divided by Square root of one plus x squared times Square root of one plus x squared divided by x minus omega squared times x divided by x plus g equals zero

to obtain the same result

x double dot times open one plus x squared close plus open g minus omega squared plus x dot above squared close times x equals zero

Further analysis on the phase plane shows that when the wire rotates not very fast ( omega less than Square root of g ) the bead oscillates around the origin in the vertical plane. If omega greater than or equals Square root of g the bead is moving along the wire away from the origin, it's velocity tending asymptotically to value Square root of omega squared minus g

Permalink 1 comment (latest comment by Niki Di Palma, Sunday, 16 Jan 2011, 08:24)
Share post
Valentin Fadeev

Descending to chaos

Visible to anyone in the world
Edited by Valentin Fadeev, Sunday, 18 Sep 2011, 23:24

The first (unguided) steps in dynamical systems. The task is to prove that the phase paths of the following system:

x double dot plus epsilon times absolute value of x times s times g times n reverse solidus full stop x dot above plus x equals zero

are isochronous spirals, that is, every circuit of every path around the origin takes the same time. It looks a bit scary, because it involves not one but two "bad" functions. Thle easiest way to deal with these is to break the picture in quadrants and examine each part separately.

equation left hand side y dot above equals right hand side negative epsilon times absolute value of x times s times g times n times y minus x

x dot above equals y

equation left hand side d times y divided by d times x equals right hand side negative epsilon times absolute value of x times s times g times n times y divided by y minus x divided by y

First quadrant: absolute value of x equals x , s times g times n times y equals one

open one plus epsilon close times x times d times x plus y times d times y equals zero

open one plus epsilon close times x squared plus y squared equals cap c

Second quadrant: equation left hand side absolute value of x equals right hand side negative x , s times g times n times y equals one

open one minus epsilon close times x times d times x plus y times d times y equals zero

open one minus epsilon close times x squared plus y squared equals cap c

Third quadrant: equation left hand side absolute value of x equals right hand side negative x , equation left hand side s times g times n times y equals right hand side negative one , the same solution as for the
first quadrant.

Fourth quadrant: absolute value of x equals x , equation left hand side s times g times n times y equals right hand side negative one , the same solution as for the
second quadrant.

Introduce polar coordinates.

I,III:

open one plus epsilon close times rho sub one squared times cosine squared of phi plus rho sub one squared times sine squared of phi equals cap c

equation sequence rho sub one squared equals cap c divided by open one plus epsilon close times cosine squared of phi plus sine squared of phi equals cap c divided by one plus epsilon times cosine squared of phi

II,IV:

open one minus epsilon close times rho sub two squared times cosine squared of phi plus rho sub two squared times sine squared of phi equals cap c

equation sequence rho sub two squared equals cap c divided by open one minus epsilon close times cosine squared of phi plus sine squared of phi equals cap c divided by one minus epsilon times cosine squared of phi

Let cap c be a closed circuit enclosing the origin. The elapsed time
is given by the following formula:

equation sequence t equals integral over open cap c close d t equals integral over open cap c close one divided by d times x divided by d times t d x equals integral over open cap c close d times x divided by x dot above equals integral over open cap c close d times x divided by y

As the functions are periodic, it is sufficient to give proof for one loop. Due to the axial symmetry of the trajectory we can calculate transit time only in the first and the second quadrants and then double the result. If we integrate counter-clockwise, then we would follow the trajectory in a reversed direction. Therefore, we need to put the negative signe before the integrals.(oh, how long it took me to realize this..) In polar coordinates this expression has the following form:

equation sequence t sub one equals negative integral over zero under pi divided by two rho sub one super prime times cosine of phi minus rho sub one times sine of phi divided by rho sub one times sine of phi d phi equals integral over zero under pi divided by two open one minus rho sub one super prime times cosine of phi divided by rho sub one times sine of phi close d phi

equation left hand side t sub two equals right hand side integral over pi divided by two under pi open one minus rho sub two super prime times cosine of phi divided by rho sub two times sine of phi close d phi

t equals two times open integral over zero under pi divided by two open one minus rho sub one super prime times cosine of phi divided by rho sub one times sine of phi close d phi plus integral over pi divided by two under pi open one minus rho sub two super prime times cosine of phi divided by rho sub two times sine of phi close d phi close

Differentiating the expression for rho sub one by phi we obtain:

equation left hand side two times rho sub one times rho sub one super prime equals right hand side negative cap c times epsilon times two times cosine of phi of negative sine of phi divided by open one plus epsilon times cosine squared of phi close squared

equation left hand side rho sub one times rho sub one super prime equals right hand side cap c times epsilon times sine of phi times cosine of phi divided by open one plus epsilon times cosine squared of phi close squared

equation sequence rho sub one super prime divided by rho sub one equals rho sub one times rho sub one super prime divided by rho sub one squared equals cap c times epsilon times sine of phi times cosine of phi divided by open one plus epsilon times cosine squared of phi close squared times one plus epsilon times cosine squared of phi divided by cap c equals epsilon times sine of phi times cosine of phi divided by one plus epsilon times cosine squared of phi

A-ha, that's where we get the independence of the final result on the path choice: the arbitrary constant gets cancelled out.

equation sequence one minus rho sub one super prime times cosine of phi divided by rho sub one times sine of phi equals one minus epsilon times sine of phi times cosine squared of phi divided by open one plus epsilon times cosine squared of phi close times sine of phi equals one minus epsilon times cosine squared of phi divided by one plus epsilon times cosine squared of phi equals one divided by one plus epsilon times cosine squared of phi

equation left hand side cap i sub one equals right hand side integral over zero under pi divided by two d times phi divided by one plus epsilon times cosine squared of phi

Let u equals tangent of phi , then equation left hand side d times phi equals right hand side d times u divided by one plus u squared , cosine squared of phi equals one divided by one plus u squared

equation sequence cap i sub one equals integral over zero under normal infinity d times u divided by open one plus epsilon divided by one plus u squared close times open one plus u squared close equals integral over zero under normal infinity d times u divided by sum with, 3 , summands one plus epsilon plus u squared equals one divided by one plus epsilon times integral over zero under normal infinity d times u divided by one plus open u divided by Square root of one plus epsilon close squared equals equals one divided by Square root of one plus epsilon times integral over zero under normal infinity d of u divided by Square root of one plus epsilon divided by one plus open u divided by Square root of one plus epsilon close squared equals one divided by Square root of one plus epsilon times arc tangent of u divided by Square root of one plus epsilon vertical line sub zero super normal infinity equals pi divided by two times one divided by Square root of one plus epsilon

equation left hand side two times rho sub two times rho sub two super prime equals right hand side negative cap c times epsilon of negative two times cosine of phi times open negative sine of phi close divided by open one minus epsilon times cosine squared of phi close squared

equation left hand side rho sub two times rho sub two super prime equals right hand side negative cap c times epsilon times sine of phi times cosine of phi divided by open one minus epsilon times cosine squared of phi close squared

equation sequence rho sub two super prime divided by rho sub two equals rho sub two times rho sub two super prime divided by rho sub two squared equals negative cap c times epsilon times sine of phi times cosine of phi divided by open one minus epsilon times cosine squared of phi close squared times one minus epsilon times cosine squared of phi divided by cap c equals negative epsilon times sine of phi times cosine of phi divided by one minus epsilon times cosine squared of phi

equation sequence one minus rho sub two super prime times cosine of phi divided by rho sub two times sine of phi equals one plus epsilon times sine of phi times cosine squared of phi divided by open one minus epsilon times cosine squared of phi close times sine of phi equals one divided by one minus epsilon times cosine squared of phi

equation sequence cap i sub two equals integral over pi divided by two under pi d times phi divided by one minus epsilon times cosine squared of phi equals integral over pi divided by two under pi d times open phi minus pi divided by two close divided by one minus epsilon times sine squared of phi minus pi divided by two equals integral over zero under pi divided by two d times psi divided by one minus epsilon times sine squared of psi d psi equals equals integral over zero under normal infinity d times u divided by open one plus u squared close times open one minus epsilon times open one minus one divided by one plus u squared close close equals integral over zero under normal infinity d times u divided by one plus open one minus epsilon close times u squared equals one divided by Square root of one minus epsilon times integral over zero under normal infinity d of u times Square root of one minus epsilon divided by one plus open u times Square root of one minus epsilon close squared equals equals one divided by Square root of one minus epsilon times arc tangent of u times Square root of one minus epsilon vertical line sub zero super normal infinity equals pi divided by two times one divided by Square root of one minus epsilon

I made some false starts on cap i sub two getting a negative answer which is impossible for a strictly positive integrand. Still not quite sure where I went wrong. Normally these things happen when a singularity sneaks in inside the domain of integration after variable change. Anyway, I decided to cheat and shift the scale.

Hence the total transit time equals:

equation sequence t equals two times open pi divided by two times one divided by Square root of one plus epsilon plus pi divided by two times one divided by Square root of one minus epsilon close equals pi times open one divided by Square root of one plus epsilon plus one divided by Square root of one minus epsilon close

Permalink Add your comment
Share post
Valentin Fadeev

Having an inte-great time

Visible to anyone in the world
Edited by Valentin Fadeev, Sunday, 16 Jan 2011, 23:18

With the new courses yet to start, hopefully providing fresh material for new posts, I have been spending time going through some excercises from new textbooks.

As integrals have always been my favourite part of calculus, I decided to take down this solution, because it just looks nice. It also illustrates the principle: don't make a substitution, until it becomes obvious.

MathJax failure: TeX parse error: Extra close brace or missing open brace

equation sequence two times x cubed minus three times x squared plus one equals two times x cubed minus two times x squared minus x squared plus one equals two times x squared times open x minus one close minus open x minus one close times open x plus one close equals open x minus one close times open two times x squared minus x minus one close equals open x minus one close times open x squared minus x plus x squared minus one close equals open x minus one close times open x times open x minus one close plus open x minus one close times open x plus one close close equals open x minus one close times open x minus one close times open two times x plus one close equals left parenthesis x minus one times right parenthesis squared times open two times x plus one close

cap i equals Square root of three times integral over negative one divided by two under zero d times x divided by Square root of left parenthesis x minus one times right parenthesis squared times open two times x plus one close

Since negative one divided by two less than or equals x less than or equals zero we have negative three divided by two less than or equals x minus one less than or equals negative one and zero less than or equals two times x plus one less than or equals one , so we need to choose negative sign when taking square root of the quadratic term.

equation sequence cap i equals Square root of three times integral over negative one divided by two under zero d times x divided by Square root of two times x plus one times open one minus x close equals Square root of three times integral over negative one divided by two under zero d of Square root of two times x plus one divided by one minus x

It's now that the substitute equation left hand side two times x plus one equals right hand side t squared becomes an obvious choice.

equation sequence cap i equals Square root of three times integral over zero under one d times t divided by one minus t squared minus one divided by two equals Square root of three times integral over zero under one d times t divided by three divided by two minus t squared divided by two equals two divided by Square root of three times integral over zero under one d times t divided by one minus t squared divided by three equals two times integral over zero under one d of t divided by Square root of three divided by one postfix minus left parenthesis t divided by Square root of three times right parenthesis squared equals two times hyperbolic tangent super negative one of t divided by Square root of three vertical line sub zero super one equals two times hyperbolic tangent super negative one of one divided by Square root of three

 

Permalink 2 comments (latest comment by Valentin Fadeev, Tuesday, 4 Jan 2011, 02:48)
Share post
Valentin Fadeev

Pfaff equation

Visible to anyone in the world
Edited by Valentin Fadeev, Wednesday, 15 Sep 2010, 23:37

Moving on with Stepanov's book I have reached the subject equations which have the following form (3 variables):

sum with, 3 , summands cap p times d times x plus cap q times d times y plus cap r times d times z equals zero

Where P, Q, R are sufficiently differentiable functions of x,y,z.

Excercise 205:

open y times z minus z squared close times d times x minus x times z times d times y plus x times y times d times z equals zero

z times open y minus z close times d times x plus x times open y times d times z minus z times d times y close equals zero

Integrating factor: mu equals one divided by y squared

z divided by y times open one minus z divided by y close times d times x plus x times y times d times z minus z times d times y divided by y squared equals zero

z divided by y times open one minus z divided by y close times d times x plus x times d of z divided by y equals zero

d times x divided by x plus d of z divided by y divided by z divided by y times open one minus z divided by y close equals zero

z divided by y equals u

d times x divided by x plus d times u divided by u minus d times u divided by u minus one equals zero

x times u divided by u minus one equals cap c

x times z divided by z minus y equals cap c

Will we always be lucky to have an appropriate factor to cast the equation into a full differential form? The book gives a negative answer setting out a very specific condition on the coefficients.

Assume the equation does have a solution and this solution is a 2-dimensional manifold, i.e has the form:

normal cap phi of x comma y comma z equals cap c

or (locally, at least):

z equals phi of x comma y

Then

equation left hand side d times z equals right hand side normal partial differential times z divided by normal partial differential times x times d times x plus normal partial differential times z divided by normal partial differential times y times d times y

On the other hand, by virtue of the equation (assuming R does not vanish identically):

equation left hand side d times z equals right hand side negative cap p divided by cap r times d times x minus cap q divided by cap r times d times y

Comparing the coefficients:

equation sequence normal partial differential times z divided by normal partial differential times x equals negative cap p divided by cap r equals cap a

equation sequence normal partial differential times z divided by normal partial differential times y equals negative cap q divided by cap r equals cap b

This is an overdetermined system: one function, two equations which generally does not have a solution The integrability condition can be obtained by equation mixed second derivatives, however I will quote the geometrical argument which may also shed light on some fact presented below.

Consider an infinitesimal shift along the manifold from open x comma y comma z close to open x plus d times x comma y close . Then z will take value:

equation left hand side z plus normal partial differential times z divided by normal partial differential times x times d times x equals right hand side z plus cap a times d times x

From here we move to the point with coordinates open x plus d times x comma y plus d times y close without leaving the manifold. New value of z:

equation left hand side sum with, 3 , summands z plus cap a times d times x plus normal partial differential divided by normal partial differential times y times open z plus cap a times d times x close times d times y equals right hand side sum with, 4 , summands z plus cap a times d times x plus cap b times d times y plus open cap a sub y plus cap a sub z times cap b close times d times x times d times y

Similarly, if we first move along y and then along x ,  we arrive at the following point:

sum with, 4 , summands z plus cap a times d times x plus cap b times d times y plus open cap b sub x plus cap b sub z times cap a close times d times x times d times y

Now we require that whatever route is chosen it leads to the same point on the manifold (up to the terms of the second order). This leads to the following equation:

cap a sub y plus cap a sub z times cap b minus cap b sub x minus cap b sub z times cap a equals zero

negative normal partial differential divided by normal partial differential times y times open cap p divided by cap r close plus normal partial differential divided by normal partial differential times z times open cap p divided by cap r close times cap q divided by cap r minus cap q divided by cap r minus normal partial differential divided by normal partial differential times z times open cap q divided by cap r close times cap p divided by cap r equals zero

equation left hand side sum with, 3 , summands cap p times open normal partial differential times cap q divided by normal partial differential times z minus normal partial differential times cap r divided by normal partial differential times z close plus cap q times open normal partial differential times cap r divided by normal partial differential times x minus normal partial differential times cap p divided by normal partial differential times z close plus cap r times open normal partial differential times cap p divided by normal partial differential times y minus normal partial differential times cap q divided by normal partial differential times x close equals right hand side zero times open one close

Now that was the book, here are some thoughts about this theory.

1) First, equation open asterisk operator close   definitely points to some categories of vector analysis. Indeed, the factors of P, Q and R are the components of the rotor of the vector field cap v right arrow of cap p comma cap q comma cap r . Hence, the condition can be rewritten in a more compact form:

cap v right arrow times r times o times t times cap v right arrow equals zero

At first sight this should hold trivially for any cap v right arrow , for the rotor is by definition perpendicular to the plane defined by cap v right arrow and tangent normal partial differential times cap v right arrow . However, this would only be true, if the solution were indeed an 2-dimensional manifold. If there is no such solution, then the whole derivation becomes invalid.

2) There is another reason why I prefer the geometric argument over comparing the mixed derivatives. The logic is very similar to that used to derive Cauchy-Riemann conditions for the analytic function. Remarkably enough, we can also apply complex formalism to the above problem. Consider the following operator:

equation left hand side diamond operator equals right hand side normal partial differential divided by normal partial differential times x plus i times normal partial differential divided by normal partial differential times y

where equation left hand side i squared equals right hand side negative one .

Assuming again that the solution exists in the form z equals phi of x comma y and using the above shortcuts for partial derivatives we obtain:

equation left hand side prefix diamond operator of z equals right hand side cap a plus i times cap b

Now apply diamond operator super asterisk operator to both parts, where asterisk operator is complex conjugate:

equation sequence diamond operator super asterisk operator diamond operator z equals open normal partial differential divided by normal partial differential times x minus i times normal partial differential divided by normal partial differential times y close times open cap a plus i times cap b close equals open normal partial differential divided by normal partial differential times x times open cap a close plus normal partial differential divided by normal partial differential times y times open cap b close close minus i times open cap a sub y plus cap a sub z times cap b minus cap b sub x minus cap b sub z times cap a close

normal partial differential divided by normal partial differential times x times open cap a close stands for "full partial derivative" where dependance of z on x is taken into account. Replacing cap a and cap b with their values, we obtain:

equation left hand side diamond operator super asterisk operator diamond operator z equals right hand side normal cap delta times z minus i times open cap a sub y plus cap a sub z times cap b minus cap b sub x minus cap b sub z times cap a close

where normal cap delta is the Laplacian, On the other hand:

equation sequence diamond operator super asterisk operator postfix diamond operator equals open normal partial differential divided by normal partial differential times x plus i times normal partial differential divided by normal partial differential times y close times open normal partial differential divided by normal partial differential times x minus i times normal partial differential divided by normal partial differential times y close equals normal cap delta

Hence

equation sequence negative normal black letter cap i times diamond operator super asterisk operator diamond operator z equals cap a sub y plus cap a sub z times cap b minus cap b sub x minus cap b sub z times cap a equals zero

which gives the above integrability condition.

So this is another example of how recourse to complex values can reveal deep facts behind the otherwise unfamiliar looking expressions. And formulate them in a nice compact form as well.

In conclusion here is an example where integrability condition does not hold:

sum with, 3 , summands d times x plus open y plus z close times d times y plus z times d times z

To solve it we rewrite it as follows:

sum with, 4 , summands d times x plus y times d times y plus z times d times z plus z times d times y equals zero

d times open x plus y squared plus z squared divided by two close plus z times d times y equals zero

Now let

x plus y squared plus z squared divided by two equals phi of y

where phi is an arbitrary function. Then

z equals negative phi super prime times open y close

These 2 relations give the general solution.

Permalink Add your comment
Share post
Valentin Fadeev

Simply complex

Visible to anyone in the world
Edited by Valentin Fadeev, Friday, 24 Sep 2010, 13:25

Discussing one excercise on the forum recently we disagreed on the point, whether turning to complex numbers makes the solution more or less straightforward.

Here I'm digging out an example showing that this tescnique is not always as obscure as it sounds. And yes, this is another example of inappropriate use of fine methods against a basic school problem:

a times sine of t plus b times cosine of t equals c

sine of t equals e super i times t minus e super negative i times t divided by two times i

cosine of equals e super i times t plus e super negative i times t divided by two

e super i times t equals z

equation left hand side a times open z minus one divided by z close plus b times i times open z plus one divided by z close equals right hand side two times c times i

open a plus b times i close times z squared minus two times c times i times z minus open a minus b times i close equals zero

cap d equals a squared plus b squared minus c squared

equation sequence z sub one comma two equals c times i plus minus Square root of cap d divided by a plus b times i equals negative b times c plus minus a times Square root of cap d divided by a squared plus b squared plus a times c minus or plus b times Square root of cap d divided by a squared plus b squared times i

equation sequence z equals e super i times t equals cosine of t plus i times sine of t

cosine of t equals negative b times c plus minus a times Square root of cap d divided by a squared plus b squared

sine of t equals a times c minus or plus b times Square root of cap d divided by a squared plus b squared

Permalink
Share post
Valentin Fadeev

Recurrencies and summing by parts

Visible to anyone in the world
Edited by Valentin Fadeev, Thursday, 22 Jul 2010, 12:23

This question was posed in sci.math.research group on Google:

http://groups.google.com/group/sci.math.research/browse_thread/thread/48ca096ec5f08a39?hl=en

After latexing the mind-bending plain text of the discussion it looks like this:

a sub one equals one

equation left hand side a sub n equals right hand side one divided by one minus b super n times n ary summation from m equals one to n minus one over a sub m times matrix row 1 n row 2 m times open one minus b close super n minus m times b super m of asterisk operator

assuming b not equals one . The author's conjecture was that for large n :

a sub n b minus one divided by natural log of b

Summing by parts is quite a standard device. Though, like with integration by parts the difficult part often is use it at the right moment. Whittakker and Watson ascribe it's systematic introduction to Abel. Probably the best account of it is given in "Concrete Mathematics". The authors introduce "definite sums" which are effectively the sums with an omitted last term:

equation left hand side n ary summation from a to b over f of x times delta times x equals right hand side n ary summation from x equals a to x equals b minus one over f of x

The cryptic delta is added solely to enhance the analogy with definite integrals.

The general formula is the following:

equation left hand side n ary summation from n equals a to n equals b minus one over u times normal cap delta times v equals right hand side u times v vertical line sub n equals a super n equals b minus n ary summation from n equals a to n equals b minus one over cap e times v times normal cap delta times u

Where equation left hand side normal cap delta times f of x equals right hand side f times open x plus one close minus f of x is the difference operator and equation left hand side cap e times f of x equals right hand side f times open x plus one close is the shift operator. The formula is easily proved by evaluating normal cap delta times u times v

equation left hand side n ary summation from a to b over normal cap delta times f of x equals right hand side f of b minus f of a

Part of the sum (*) on the right prompts for the binomial formula. Hence, it would be good to pull it out of the sum. Let's try:

equation sequence u equals a sub m times normal cap delta times v equals matrix row 1 n row 2 m times open one minus b close super n minus m times b super m

equation left hand side normal cap delta times u equals right hand side a sub m plus one minus a sub m

equation sequence v equals n ary summation from m equals one to x over matrix row 1 n row 2 m times open one minus b close super n minus m times b super m vertical line sub one super n equals one postfix minus left parenthesis one minus b times right parenthesis super n minus n times b left parenthesis one minus b times right parenthesis super n minus one

For

equation left hand side n ary summation from m equals one to n over matrix row 1 n row 2 m times open one minus b close super n minus m times b super m equals right hand side

equation left hand side equals right hand side n ary summation from m equals zero to n over open matrix row 1 n row 2 m times open one minus b close super n minus m times b super m close postfix minus left parenthesis equation left hand side one minus b times right parenthesis super n equals right hand side left parenthesis one minus b plus b times right parenthesis super n postfix minus left parenthesis equation left hand side one minus b times right parenthesis super n equals right hand side one postfix minus left parenthesis one minus b times right parenthesis super n

Putting it all together:

equation left hand side n ary summation from m equals one to n minus one over a sub m times matrix row 1 n row 2 m times open one minus b close super n minus m times b super m equals right hand side

equation left hand side equals right hand side a sub n left parenthesis one postfix minus left parenthesis one minus b times right parenthesis super n minus n times b times open one minus b times right parenthesis super n minus one close minus n ary summation from m equals one to n minus one over open a sub m plus one minus a sub m close left parenthesis one postfix minus left parenthesis equation left hand side one minus b times right parenthesis super n minus n times b times open one minus b times right parenthesis super n minus one close equals right hand side

equation left hand side equals right hand side a sub n left parenthesis one postfix minus left parenthesis one minus b times right parenthesis super n minus n times b times open one minus b times right parenthesis super n minus one close postfix minus left parenthesis one postfix minus left parenthesis equation left hand side one minus b times right parenthesis super n minus n times b times open one minus b times right parenthesis super n minus one close times open a sub n minus one close equals right hand side one postfix minus left parenthesis one minus b times right parenthesis super n minus n times b left parenthesis one minus b times right parenthesis super n minus one

This gives an equation for a sub n :

equation left hand side a sub n times open one minus b super n close equals right hand side one postfix minus left parenthesis one minus b times right parenthesis super n minus n times b left parenthesis one minus b times right parenthesis super n minus one

equation left hand side a sub n equals right hand side one postfix minus left parenthesis one minus b times right parenthesis super n minus n times b left parenthesis one minus b times right parenthesis super n minus one divided by one minus b super n

Permalink 2 comments (latest comment by Valentin Fadeev, Sunday, 25 Jul 2010, 21:50)
Share post
Valentin Fadeev

Gunter's example

Visible to anyone in the world
Edited by Valentin Fadeev, Tuesday, 6 Jul 2010, 23:04

This one gave me some hard time:

z times open x plus z close times normal partial differential times z divided by normal partial differential times x minus y times open y plus z close times normal partial differential times z divided by normal partial differential times y equals zero

where z equals Square root of y when x equals one

The difference with "ordinary" linear equations is that coefficients here depend on z . At the same time right hand side is identically zero, so this is not strictly an inhomogeneous equations.

I made false starts trying to find some nice substitute to absorb z or alternatively pull out the missing term to construct an inhomogeneous equations. Finally, I found a hint in the book N.M. Günter, Integration of First-Order Partial Differential Equations, ONTI/GTTI, Leningrad/Moscow (1934). In the solution for an equation of a similar structure it was suggested to use the standard method and treat z as a constant when integrating the associated system.

So here we go. Searching for a solution in an implicit form:

cap v of z comma x comma y equals zero

z times open x plus z close times normal partial differential times cap v divided by normal partial differential times x minus y times open y plus z close times normal partial differential times cap v divided by normal partial differential times y equals zero

The associated system:

equation sequence d times x divided by z times open x plus z close equals negative d times y divided by y plus z equals d times z divided by zero equals d times t

In the same book it is hinted that one particular integral of this system is z equals cap c sub one which follows from the last identity. Another integral can be found using the first identity:

d times x divided by z times open x plus z close plus d times y divided by y plus z equals zero

Treating z as constant we can simplify the expressions:

d of x divided by z divided by one plus x divided by z plus d of y divided by z divided by y divided by z times open one plus y divided by z close equals zero

equation sequence x divided by z equals u times y divided by z equals v

d times u divided by one plus u plus d times v divided by v minus d times v divided by one plus v equals zero

equation left hand side u times open v plus one close divided by v plus one equals right hand side cap c sub two

equation left hand side open x plus z close times y divided by y plus z equals right hand side cap c sub two

Therefore the general solution can be written in the form:

equation sequence cap v of z comma x comma y equals normal cap phi of z comma open x plus z close times y divided by y plus z equals zero

It is already within reach of sheer guess to let normal cap phi of u comma v equals u minus v to establish the result, however we proceed with a more lengthy, yet rigorous way.

The system of the first integrals is written as follows:

psi sub one equals z

equation left hand side psi sub two equals right hand side open x plus z close times y divided by y plus z

Using the initial condition x equals one :

psi one macron equals z

equation left hand side psi two macron equals right hand side open one plus z close times y divided by y plus z

Solving for z and y :

z equals psi one macron

equation sequence y equals z times psi two macron divided by one plus z minus psi two macron equals psi one macron times psi two macron divided by one plus psi one macron minus psi two macron

Now following the standard method already mentioned below:

MathJax failure: TeX parse error: Extra open brace or missing close brace

equation left hand side psi sub one equals right hand side psi sub two divided by one plus psi sub one minus psi sub two

z equals open x plus z close times y divided by y plus z divided by one plus z minus open x plus z close times y divided by y plus z

equation left hand side sum with, 3 , summands z times y plus z squared plus open z squared minus x times y close times z equals right hand side x times y plus z times y

open z squared minus x times y close times open z plus one close equals zero

Suppressing the solution z equals negative one which does not satisfy intial conditions, finally we obtain:

z equals Square root of x times y

 

Permalink
Share post
Valentin Fadeev

Going further

Visible to anyone in the world
Edited by Valentin Fadeev, Monday, 21 Jun 2010, 19:59

Strolling struggling on:

equation left hand side open m times z minus n times y close times normal partial differential times z divided by normal partial differential times x plus open n times x minus l times z close times normal partial differential times z divided by normal partial differential times y equals right hand side l times y minus m times x

This is an inhomogeneous equation. Following the theory we try to
find the general solution in an implicit form:

cap v of z comma x comma y equals zero

It is proven that the solution found in this form is indeed
general, i.e. we are not losing any solutions on the way.

equation left hand side normal partial differential times z divided by normal partial differential times x equals right hand side negative normal partial differential times cap v divided by normal partial differential times x divided by normal partial differential times cap v divided by normal partial differential times z

equation left hand side normal partial differential times z divided by normal partial differential times y equals right hand side negative normal partial differential times cap v divided by normal partial differential times y divided by normal partial differential times cap v divided by normal partial differential times z

sum with, 3 , summands open m times z minus n times y close times normal partial differential times cap v divided by normal partial differential times x plus open n times x minus l times z close times normal partial differential times cap v divided by normal partial differential times y plus open l times y minus m times x close times normal partial differential times cap v divided by normal partial differential times z equals zero

Now we can write the associated system (in the symmetrical form):

equation sequence d times x divided by m times z minus n times y equals d times y divided by n times x minus l times z equals d times z divided by l times y minus m times x equals d times t

Or more conveniently for this example, in the canonical form:

equation left hand side d times x divided by d times t equals right hand side m times z minus n times y

equation left hand side d times y divided by d times t equals right hand side n times x minus l times z

equation left hand side d times z divided by d times t equals right hand side l times y minus m times x

Multiplying these equations by l, m and n respectively and summing we get:

d times open sum with, 3 , summands l times x plus m times y plus n times z close divided by d times t equals zero

equation left hand side sum with, 3 , summands l times x plus m times y plus n times z equals right hand side cap c sub one

This is one of the first integrals of the system. Now multiplying the equations by x, y and z respectively amd summing we obtain:

one divided by two times d times open sum with, 3 , summands x squared plus y squared plus z squared close divided by d times t equals zero

equation left hand side sum with, 3 , summands x squared plus y squared plus z squared equals right hand side cap c sub two

Therefore, the general solution has the following form:

normal cap phi of sum with, 3 , summands l times x plus m times y plus n times z comma sum with, 3 , summands x squared plus y squared plus z squared equals zero

Geometrically the first solution represents a plane in a 3d space with angular coefficients of the normal vector cosine of alpha colon cosine of beta colon cosine of gamma equals l colon m colon n . The second integral represents a sphere centered at the origin. Therefore, the characteristics of the equation (the curves,

resulting from intersection of these surfaces) are the circles centered on the line passing through the origin with the above mentioned angular coefficients.

Indeed, another way to look at it is rewrite the equation in the following form:

$$\left|\begin{array}{ccc}

\frac{\partial u}{\partial x} & \frac{\partial u}{\partial y} & \frac{\partial u}{\partial z}\\

l & m & n\\

x & y & z\end{array}\right|=0$$

Or:

tau right arrow dot operator open n right arrow multiplication r right arrow close equals zero

Where tau right arrow is a tangential vector to the surface u of x comma y comma z equals zero , n right arrow is the vector of the axis of revolution and r right arrow is the radius vector of an arbitrary point on the surface. It means that for every point on the surface the tangent vector must lie in the plane

passing through the axis of revolution. This is natural, for the surface is ontained by rotating a plane curve against the axis.

Permalink
Share post
Valentin Fadeev

Going partial

Visible to anyone in the world
Edited by Valentin Fadeev, Friday, 18 Oct 2019, 07:52

Struggling through the early chapters of partial differential equations (from the all-time-classic Stepanov's book) I often run across the moves that are not really technically demanding, but the logic behind takes time to sink in.

One of them is the trick used to find a particular solution of the first order linear PDE satisfying initial condition:

f of x sub one comma x sub two comma full stop full stop full stop comma x sub n minus one comma x sub n super zero equals phi of x sub one comma x sub two comma full stop full stop full stop comma x sub n minus one

Here is an excercise: find the solution of the following equation

sum with, 3 , summands Square root of x times normal partial differential times f divided by normal partial differential times x plus Square root of y times normal partial differential times f divided by normal partial differential times x plus Square root of z times normal partial differential times f divided by normal partial differential times z equals zero

         satisfying the condition: f equals y minus z when x equals one

We start by writing out the associated system of ODEs:

equation sequence d times x divided by Square root of x equals d times y divided by Square root of y equals d times z divided by Square root of z

Use the first identity to find a "first integral" of the system:

equation left hand side d times x divided by Square root of x equals right hand side d times y divided by Square root of y

equation sequence psi sub one equals Square root of x minus Square root of y equals cap c sub one

Then equate the first and the third quotients to obtain the second "first integral":

equation left hand side d times x divided by Square root of x equals right hand side d times z divided by Square root of z

equation sequence psi sub two equals Square root of x minus Square root of z equals cap c sub two

Obviously there are no more independent first integrals

According to the theory, the general solution is given as an arbitrary function of these two integrals:

equation sequence f equals normal cap phi of psi sub one comma psi sub two equals normal cap phi of Square root of x minus Square root of y comma Square root of x minus Square root of z

Now we shall find the particular solution. Let x=1. Following the book, we introduce new functions psi one macron and psi two macron to which psi sub one and psi sub two turn when we set the value of x :

equation left hand side psi one macron equals right hand side one minus Square root of y

equation left hand side psi two macron equals right hand side one minus Square root of z

Now we solve these equations with respect to y and z :

y equals open one minus psi macron sub one close squared times open one close

z equals open one minus psi two macron close squared

And now comes the difficult part. In order to get the final result we need to substitute the above results in the expression of f replacing psi macron with psi :

equation sequence f equals y minus z equals open one minus psi sub one close squared minus open one minus psi sub two close squared equals psi sub one squared minus psi sub two squared plus two times open psi sub two minus psi sub one close

equation sequence f equals x minus two times Square root of x times y plus y minus x plus two times Square root of x times z minus z plus two times open Square root of y minus Square root of z close equals y minus z plus two times open Square root of x plus one close times open Square root of z minus Square root of y close

It took me some time to accept that I have to substitute psi which is a more general expression than  psi macron in the equation which was derived after giving x ist particular value.

Finally:

f equals y minus z plus two times open Square root of x plus one close times open Square root of z minus Square root of y close

The reasoning is clear, once it is explained.  psi sub one and psi sub two are both solutions, hence the function of them is also a solution. Then, if we let x equals one they turn to  psi one macron and psi two macron . Then by virtue of the equations (1) we get the variables y and z and consequently the expression of phi of y comma z as required.

In the book the whole argument is presented in the most general form. However, it takes several excercises and hours of thinking to get a hands-on experience with this method.

Permalink
Share post
Valentin Fadeev

Calculus of Warehouses. Part 4

Visible to anyone in the world
Edited by Valentin Fadeev, Thursday, 17 Jun 2010, 22:02

I am posting it in reply to a question I came across in one social network. There is no original material here. I am just trying to reproduce the argument from one university lecture, as far as I can remember it.

The task is to calculate optimal stock level at a warehouse. Stock is replenished to a certain value (which is to be determined) immediately at fixed equally spaced moments in time. Amount of stock units cnosumed during each period is random.

Let s be the stock level which is maintained. Assume that stock is replenished over a fixed period of time (take it to be unitary to simplyfy calculations). Assume that r units are used during each time period with probability cap p of r .

Let cap c sub one be unitary storage cost, cap c sub two is a “penalty” cost
assigned when stock is theoretically below 0, or below some
minimal admissible level (lost orders, emergency orders, etc).

Assume also that the stock is replenished immediately.

Consider two possible situations.

1eaa1f85b6c049d9e84d4eb8cc732c16.jpg

1) r less than or equals s , i.e there is a positive remainder in the warehouse at
the end of the period. Storage cost is proportional to the area
under the stock graph.

equation sequence cap e sub one super r of s equals cap c sub one times s plus open s minus r close divided by two equals cap c sub one times open s minus r divided by two close

Expectancy of this value is:

cap m of cap e sub one super r of s equals cap c sub one times cap p of r times open s minus r divided by two close

Therefore the average cost is:

cap e sub one of s equals cap c sub one times n ary summation from r equals zero to s over cap p of r times open s minus r divided by two close

2) r greater than s . In this case the cost of both types is incurred. Again using the geometrical approach we find that cost elements are proportional to the areas of the triangles. The same argument as above gives the answer:

cap e sub two of s equals cap c sub one times n ary summation from r equals s plus one to normal infinity over cap p of r times s squared divided by two times r plus cap c sub two times n ary summation from r equals s plus one to normal infinity over cap p of r times open r minus s close squared divided by two times r

Thus the total cost is given by the following expression:

cap e of s equals sum with, 3 , summands cap c sub one times n ary summation from r equals zero to s over cap p of r times open s minus r divided by two close plus cap c sub one times n ary summation from r equals s plus one to normal infinity over cap p of r times s squared divided by two times r plus cap c sub two times n ary summation from r equals s plus one to normal infinity over cap p of r times open r minus s close squared divided by two times r

Marginal analysis is used to determine the optimal stock level. Optimal value of s minimizing cap e satisfies the relation:

cap e of s less than cap c sub one divided by cap c sub one plus cap c sub two less than cap e times open s plus one close

(here my memory is abit vague, so I am giving it without proof and without guarantee)

Practically this means calculating cap e of s for a series of the values of s until the above double inequality holds.

Permalink
Share post
Valentin Fadeev

All around the Jacobi equation

Visible to anyone in the world
Edited by Valentin Fadeev, Sunday, 18 Sep 2011, 23:25

The Jacobi equation for a functional serving as a test for weak local extremum can be derived in quite different ways. The following geometrical approach was found in the book "Calculus of Variations" by L.E. Elsgoltz, 1958. It hangs upon the question of whether or not the stationary path can be included in a part of a one-parametric family of stationary paths (field of paths). There can be 2 options. Either no 2 paths of the family intersect, or all patghs of the family share 1 common point (but not more) in the given interval.

For example y equals cap c times sine of x will form a family of the first type in open delta comma a close , where delta greater than zero , a less than pi , the family of the second type in open zero comma a close , a less than pi . In the interval open zero comma a close , a greater than or equals pi no such family can be constructed.

Suppose we have a one-parametric family of stationary paths y equals y of x comma cap c . For example, we can fix one of the boundary points and use the gradient of the paths in this point as parameter C.

paths

The envelope of this family is found by eliminating C from the following system of equations:

equation sequence y equals y of x comma cap c times normal partial differential times y of x comma cap c divided by normal partial differential times cap c equals zero times open asterisk operator close

Along each path of the family normal partial differential times y of x comma cap c divided by normal partial differential times cap c is a function of only x. Denote this function as u of x for some given C. Then equation left hand side u sub x super prime equals right hand side normal partial differential squared times y of x comma cap c divided by normal partial differential times cap c times normal partial differential times x .

y equals y of x comma cap c are solutions of the E-L by assumption. Therefore:

cap f sub y of x comma y of x comma cap c comma y sub x super prime of x comma cap c minus d divided by d times x times cap f sub y super prime of x comma y of x comma cap c comma y sub x super prime of x comma cap c equals zero

Differentiating this equality by C and letting u equals normal partial differential times y of x comma cap c divided by normal partial differential times cap c we obtain:

cap f sub y times y times u plus cap f sub y times y super prime times u super prime minus d divided by d times x times open cap f sub y times y super prime times u plus cap f sub y super prime times y super prime times u super prime close equals zero

Rearranging we get:

open cap f sub y times y minus d divided by d times x times cap f sub y times y super prime close times u minus d divided by d times x times open cap f sub y super prime times y super prime times u super prime close equals zero

which is obviously a Jacobi equation.

Thus, if u has a zero somewhere in the interval, it follows from (*) above that this is a common point of the stationary path and the envelope. This is a point, conjugate to the left end of the interval.

It seemed to me at first that this proof serves only for theoretical purpose, as another way of deriving the Jacobi equation. However, the idea behind can be used to find the solution of the Jacobi equation, without actually solving the equation itself!

Consider the following example.

equation sequence cap s of y equals integral over zero under one y super prime two divided by y super four d x times y of zero equals one semicolon y of one equals one divided by two

equation left hand side y super prime times two times y super prime divided by y super four minus y super prime two divided by y super four equals right hand side cap c squared

y super prime divided by y squared equals cap c

equation left hand side negative one divided by y equals right hand side cap c times x plus cap c sub zero

(we can suppress plus minus , assuming it is absorbed by the constant)

y equals one divided by cap c sub one times x plus cap c sub two

Now apply boundary conditions:

equation sequence y of zero equals one times y of one equals one divided by two

y equals one divided by x plus one

If we only use the first condition, that is fix the left boundary, we get the one-parametric family:

y of x comma cap c equals one divided by cap c times x plus one

C being the gradient at x equals zero with a reverse sign. Now following the idea described above we can find u of x :

equation sequence u of x equals normal partial differential times y of x comma cap c divided by normal partial differential times cap c equals negative x divided by open cap c times x plus one close squared

Finally setting cap c equals one by virtue of the boundary conditions:

u of x equals negative x divided by open x plus one close squared

Now we move on to dervie the Jacobi equation through the coefficients of the second variation:

equation sequence cap p of x equals normal partial differential squared times cap f divided by normal partial differential times y super prime two equals two divided by y super four equals two left parenthesis x plus one times right parenthesis super four

equation sequence cap q of x equals normal partial differential squared times cap f divided by normal partial differential times y squared minus d divided by d times x times open normal partial differential squared times cap f divided by normal partial differential times y times normal partial differential times y super prime close equals 20 times y super prime two divided by y super six plus eight times d divided by d times x times open y super prime divided by y super five close equals 20 times y super prime two divided by y super six plus eight times y super double prime divided by y super five minus 40 times y super prime two divided by y super six equals eight times y super double prime divided by y super five minus 20 times y super prime two divided by y super six

equation sequence cap q of x equals 16 times open x plus one close super five divided by open x plus one close cubed minus 20 times open x plus one close super six divided by open x plus one close super four equals negative four times open x plus one close squared

Inserting the above results into the equation:

left parenthesis cap p times u super prime times right parenthesis super prime minus cap q times u equals zero

two times open open x plus one close super four times u super prime close super prime plus four times open x plus one close squared times u equals zero

open open x plus one close super four times u super prime close super prime plus two times open x plus one close squared times u equals zero

sum with, 3 , summands open x plus one close super four times u super double prime plus four times open x plus one close cubed times u super prime plus two left parenthesis x plus one times right parenthesis squared times u equals zero

sum with, 3 , summands u super double prime plus four divided by x plus one times u super prime plus two divided by open x plus one close squared times u equals zero

Instead of solving the equation which can be technically demanding, we shall verify if the expression for u of x found above is a solution. We do not need a general solution in this case. All non-trivial solutions of a homogeneous equation of the second order satisfying the condition u of x sub zero equals zero differ from each other only by a constant multiplier and thus have the same zeros.

u equals negative x divided by open x plus one close squared

equation sequence u super prime equals negative one divided by open x plus one close squared plus two times x divided by open x plus one close cubed equals negative x minus one plus two times x divided by open x plus one close cubed equals x minus one divided by open x plus one close cubed

equation sequence u super double prime equals one divided by open x plus one close cubed minus three times x minus one divided by open x plus one close super four equals x plus one minus three times x plus three divided by open x plus one close super four equals negative two times x minus two divided by open x plus one close super four

negative two times x plus two divided by open x plus one close super four times open x plus one close super four plus four times x minus one divided by open x plus one close cubed times open x plus one close cubed minus two times x divided by open x plus one close squared times open x plus one close squared equals zero

sum with, 3 , summands negative two times x plus four plus four times x minus four minus two times x equals zero

So we indeed have a solution.

For the integrand does not depend explicitly on x we can simplify the Jacobi equation by exchanging the roles of the variables:

equation sequence cap s of y equals integral over zero under one y super prime two divided by y super four d x equals integral over zero under one open d times y close squared divided by open d times x close squared times one divided by y super four d x equals integral over one under one divided by two one divided by x super prime times y super four d y equals negative integral over one divided by two under one one divided by x super prime times y super four d y equals cap s of x

cap g of y comma x comma x super prime equals negative one divided by x super prime times y super four

Euler-Lagrange equation has the first integral:L

cap g sub x super prime equals cap c

equation left hand side negative one divided by x super prime two times y super four equals right hand side negative c squared

x equals cap c sub one divided by y plus cap c sub two

cap c sub one plus cap c sub two equals zero

Again we leave one arbitrary constant to form a family:

x of y comma cap c equals cap c times open one divided by y minus one close

equation sequence u of y equals normal partial differential times x of y comma cap c divided by normal partial differential times cap c equals one divided by y minus one

equation sequence cap p of y equals normal partial differential squared times cap g divided by normal partial differential times x super prime two equals negative two divided by x super prime three times y super four equals negative two divided by negative one divided by y super six times y super four equals two times y squared

Since the transformed integrand does not depend explicitly on the dependent variable, Q will vanish and the Jacobi equation has the first integral:

cap p times u sub y super prime equals cap c

equation left hand side two times y squared times u sub y super prime equals right hand side negative two times cap c sub one

u of y equals cap c sub one divided by y plus cap c sub two

cap c sub one plus cap c sub two equals zero

equation left hand side cap c sub one equals right hand side negative one

u of y equals one minus one divided by y
Thus, up to the sign we get the same expression. (I am not sure where I may be loosing the sign, but it obviously has litle effect on the argument)

Permalink Add your comment
Share post
Valentin Fadeev

Lemniscate functions: the lost symbols

Visible to anyone in the world
Edited by Valentin Fadeev, Sunday, 18 Sep 2011, 23:25

The first time I encountered these weird objects of analysis was probably while surfing the book of E. Kamke "A reference of ordinary differential equations" which in turn gave a reference to Whittakker and Watson. I was at that time delving into the methods of analytic geometry and only new that lemniscate was an 8-shaped algebraic curve of the 4th order, a particular case of Cassini ovals. So I was pretty shocked to find out that it could give rise to some "trigonometric" system.

Although these functions were already studied by Gauss (no surprise), most of the original and subsequent research concentrated on different series expansions and evaluation of particular elliptic integrals.

Being fresh from the first course on calculus, I attempted an investigation of the properties of lemniscate functions by means of only the very basic techniques used derive similar results for circular functions. I wanted to derive formulas for derivatives and primitives, addition theorems, complementary formulas, etc.

However, looking back at that paper I see that in the most crucial steps I just took the known relations for Jacobi elliptic functions (from W&W book) and then reduced them to the particular case of lemniscate functions.

I think that lemniscate functions can be used for changing variable when evaluating certain integrals, or converting differential equations into manageable forms. Although in my rather limited practice I have never encountered the cases where it would be appropriate, I still want to make a small, but independent account of these devices and keep them in my arsenal waiting for the right moment to come.

So, enough of the words, let's get down to business.

Lemniscate functions arise when rectifying the length of lemniscate and are defined by inversion of an integral (see http://mathworld.wolfram.com/LemniscateFunction.html for intermediate steps):

equation sequence phi equals integral over zero under s times l times phi d times t divided by Square root of one minus t super four times phi equals integral over c times l times phi under one d times t divided by Square root of one minus t super four

(I am using the original Gaussian (and my own ;)) notation instead of the more lengthy sinlemn and coslemn)

Take the first integral and differentiate both sides by phi :

MathJax failure: TeX parse error: Extra open brace or missing close brace

equation left hand side s times l super prime times phi equals right hand side Square root of one minus s times l super four times phi

equation left hand side s times l super prime two times phi equals right hand side one minus s times l super four times phi

equation left hand side two times s times l super prime times phi times s times l super double prime times phi equals right hand side negative four times s times l cubed times phi times s times l super prime times phi

equation left hand side s times l super double prime times phi equals right hand side negative two times s times l cubed times phi

The same would hold, if we started with the second integral. Hence we obtain the differential equation for lemniscate functions:

equation left hand side y super double prime equals right hand side negative two times y cubed

Now we shall find an algebraic relation between sl and cl.

equation sequence phi equals integral over c times l times phi under one d times t divided by Square root of one minus t super four equals integral over c times l times phi under one d times t divided by Square root of one minus t squared divided by one plus t squared times open one plus t squared close

Substitute equation left hand side t squared equals right hand side one minus z squared divided by one plus z squared . Then

equation left hand side two times t times d times t equals right hand side negative two times z times open one plus z squared close minus two times z times open one minus z squared close divided by left parenthesis one plus z squared times right parenthesis squared times d times z

equation left hand side t times d times t equals right hand side negative two times z cubed times d times x divided by left parenthesis one plus z squared times right parenthesis squared

equation left hand side d times t equals right hand side negative Square root of one plus z squared divided by one minus z squared times two times z cubed times d times z divided by left parenthesis one plus z squared times right parenthesis squared

Inserting it all in the integral and observing new integration limits we obtain:

equation sequence phi equals negative integral over Square root of one minus c times l squared times phi divided by one plus c times l squared times phi under zero one divided by z times open one plus z squared close divided by two times z squared times Square root of one plus z squared divided by one minus z squared times two times z cubed times d times z divided by left parenthesis one plus z squared times right parenthesis squared equals integral over zero under Square root of one minus c times l squared times phi divided by one plus c times l squared times phi d times z divided by Square root of one minus z squared divided by one plus z squared times open one plus z squared close equals integral over zero under Square root of one minus c times l squared times phi divided by one plus c times l squared times phi d times z divided by Square root of one minus z super four equals integral over zero under Square root of one minus c times l squared times phi divided by one plus c times l squared times phi d times t divided by Square root of one minus t super four

Now comparing this with the integral defining s times l times phi and looking at the limits we conclude that:

equation left hand side s times l squared times phi equals right hand side one minus c times l squared times phi divided by one plus c times l squared times phi

Conversely:

equation left hand side c times l squared times phi equals right hand side one minus s times l squared times phi divided by one plus s times l squared times phi

Now it is easy to establish the expressions for derivatives:

equation sequence s times l super prime times phi equals Square root of one minus s times l super four times phi equals Square root of one minus s times l squared times phi divided by one plus s times l squared times phi times open one plus s times l squared times phi close equals c times l times phi times open one plus s times l squared times phi close

Formula for cl can be obtained similarly, but we can follow a different path using complimentary formula.

Rewrite the definitions in the following way:

equation sequence phi equals s times l super negative one times x equals integral over zero under x d times t divided by Square root of one minus t super four times phi macron equals c times l super negative one times x equals integral over x under one d times t divided by Square root of one minus t super four

Then:

equation sequence phi plus phi macron equals s times l super negative one times x plus c times l super negative one times x equals integral over zero under one d times t divided by Square root of one minus t super four equals c times o times n times s times t

x equals s times l times phi

x equals c times l times phi macron

So

equation left hand side s times l times phi equals right hand side c times l times open cap c minus phi close

Then we immediately obtain:

equation left hand side c times l super prime times phi equals right hand side negative s times l times phi times open one plus c times l squared times phi close

The constant value which is half the length of the "unitary" lemniscate can be evaluated substituting one minus t super four equals u in the integral:

cap c equals negative one divided by four times integral over one under zero u super negative one divided by two left parenthesis equation left hand side one minus u times right parenthesis super negative three divided by four times d times u equals right hand side one divided by four times integral over zero under one u super negative one divided by two left parenthesis equation sequence one minus u times right parenthesis super negative three divided by four times d times u equals one divided by four times cap b of one divided by two comma one divided by four equals one divided by four times normal cap gamma of one divided by two times normal cap gamma of one divided by four divided by normal cap gamma of three divided by four equals left square bracket normal cap gamma of one divided by four times right square bracket squared divided by four times Square root of two times Square root of pi

Integral of the lemniscate function is easily calculated:

equation sequence integral s times l times phi d phi equals integral t times d of s times l super negative one times t equals integral t times d times t divided by Square root of one minus t super four equals one divided by two times integral d of t squared divided by Square root of one minus open t squared times right parenthesis squared close equals arc sine of t squared divided by two equals arc sine of s times l squared times phi divided by two

Now I am only one step away from deriving the addition theorem using the same method due to Euler that works for Jacobi elliptic functions, but got somewhat lost in the algebra.. Hope to post that later on.

Permalink Add your comment
Share post
Valentin Fadeev

Return of the recurrencies

Visible to anyone in the world
Edited by Valentin Fadeev, Thursday, 15 Apr 2010, 00:21

I have always been amazed by the power of recurrent formulas. Although they are likely to cause stack overflow (this was the first occasion when i learned what it actually meanssmile), used carefully they deliver beautiful results.

I once amused myself drawing stellar polygons of arbitrary number of vertices. That required a simple parametrized procedure that produced a sequence of vertices to be connected with line segments.

Starting with the formula of the n-th root of complex number (of unitary modulus):

MathJax failure: TeX parse error: Extra open brace or missing close brace

to get the coordinates of the n vertices, i found the following way to produce the path sequence:

equation left hand side v sub k equals right hand side open v sub k minus one plus d close mod n

where v sub k is the number of the vertex and d ( one less than or equals d less than or equals open n divided by two close ) is the number of vertices "skipped" in one step.

So the procedure looks roughly like this:

 

x0=cos(phi/n)

y0=sin(phi/n)

do

vn=(v+d) mod n

x1=cos((phi+2*pi*vn)/n)

y1=sin((phi+2*pi*vn)/n)

line(x0,y0)-(x1,y1)

v=vn

x0=x1

y0=y1

loop until vn=0

I even considered the idea of enumerating the possible outcomes for different pairs of (n,d), but put it on the shelf then..

Permalink
Share post
Valentin Fadeev

Facts about integrating factors

Visible to anyone in the world
Edited by Valentin Fadeev, Sunday, 18 Sep 2011, 23:26

Even if you are faced with a plain separable ODE, the process of separation of variables itself implies multiplying both parts by some factor. Thus the integrating factor seems to be one of the most devious tricks of solving equations.

There is a general path to establish its existence. It can be found in many textbooks. I am interested in some particular cases here which give beautiful solutions.

First, for a homogeneous equation it is possible to find a closed formula for the integrating factor.

It can be shown that for equation

cap m of x comma y times d times x plus cap n of x comma y times d times y equals zero ,

where M and N are homogeneous functions of their arguments integrating factor has the form:

mu equals one divided by x times cap m of x comma y plus y times cap n of x comma y

Apply this to equation:

open a squared times x minus y close times d times x plus open x plus y close times d times y equals zero

equation sequence mu equals one divided by x times open a squared times x minus y close plus y times open x plus y close equals one divided by a squared times x squared plus y squared

Multiplying both parts by this expression we obtain:

a squared times x minus y divided by a squared times x squared plus y squared times d times x plus x plus y divided by a squared times x squared plus y squared times d times y equals zero

Rearranging:

x times d times y minus y times d times x divided by a squared times x squared plus y squared plus a squared times x times d times x plus y times d times y divided by a squared times x squared plus y squared equals zero

d of x divided by y divided by a squared postfix plus left parenthesis x divided by y times right parenthesis squared plus one divided by two times d times open a squared times x squared plus y squared close divided by a squared times x squared plus y squared equals zero

And the result becomes obvious.

For the next example it is useful to note the fact that if mu is an integrating factor for equation cap m times d times x plus cap n times d times y equals zero giving solution in the form cap u of x comma y equals cap c , then equation left hand side mu sub one equals right hand side mu times phi of cap u where phi is any differentiable function shall also be an integrating factor. Indeed

equation sequence mu sub one times open cap m times d times x plus cap n times d times y close equals phi of cap u times mu times open cap m times d times x plus cap n times d times y close equals phi of cap u times d times cap u

giving the differential for the function normal cap phi of cap u equals integral phi of cap u d cap u

This leads to the following practical trick of finding the factor. All terms of the equations are split in two groups for each of which it is easy to find the integrating factor. Then each factor is written in the most general form involving an arbitrary function as described above. Finally we try to find such functions that make both factors identical.

Consider the following equation:

open x squared times y squared minus one close times d times y plus two times x times y cubed times d times x equals zero

Rearranging the terms:

open x squared times y squared times d times y plus two times x times y cubed times d times x close minus d times y equals zero

For the second term now the integrating factor is trivial, it is 1. Hence the most general form will look like mu sub one equals phi of y .

For the first part it is easy to see that the factor should be one divided by x squared times y cubed giving solution x squared times y equals cap c , hence equation left hand side mu sub two equals right hand side one divided by x squared times y cubed times psi of x squared times y .

To make the two identical we want mu sub two to be independent of x. Setting psi of t equals t gives equation left hand side mu sub two equals right hand side one divided by y squared .

Applying this one we get:

open x squared times d times y plus two times x times y times d times x close minus d times y divided by y squared equals zero

x squared times y plus one divided by y equals cap c

Both methods were discovered in the classic book "A course on differential equations" by V.V. Stepanov

Permalink 2 comments (latest comment by Valentin Fadeev, Thursday, 15 Apr 2010, 16:02)
Share post
Valentin Fadeev

Big O-Oh

Visible to anyone in the world
Edited by Valentin Fadeev, Sunday, 16 Jan 2011, 23:17

There are many good articles on O symbols, some more technical, some more popular. But at the end of the day you often find yourself staring at the excercise remembering all those definitions and still not knowing what to do next. This has been the case for me, until i took some freedom to play around with this device. I think i finally got it down. Here are two examples:

x1/2/(1+O(x))=x1/2(1+O(x))=x1/2+x1/2O(x)=O(x1/2)+O(x3/2)

=O(x1/2)+O(x1/2)=2O(x1/2)=O(x1/2)

Where in the first equality i used geometric expansion until the linear term and the fact that for non-zero k kO(x)=O(x), hence -O(x)=O(x). Of course "=" sign must be understood as element of through all manipulations with O.

A more demanding one:

equation sequence two times phi minus sine of two times phi divided by two times sine squared of phi equals sum with, 3 , summands two times phi minus two times phi plus eight divided by six times phi cubed plus cap o of phi super five divided by two left parenthesis phi plus cap o of phi cubed times right parenthesis squared equals four divided by three times phi cubed plus cap o of phi super five divided by two times open sum with, 3 , summands phi squared plus cap o of phi super four plus cap o of phi super six close equals

equation sequence equals four divided by three times phi cubed plus cap o of phi super five divided by two times open phi squared plus cap o of phi super four close equals two divided by three times phi times one plus cap o of phi squared divided by one plus cap o of phi squared right parenthesis equals two divided by three times phi times open one plus cap o of phi squared close times open one plus cap o of phi squared close equals

equation sequence equals two divided by three times phi times open sum with, 3 , summands one plus cap o of phi squared plus cap o of phi super four close equals two divided by three times phi times open one plus cap o of phi squared close equals two divided by three times phi plus cap o of phi cubed

Of course, it would be a bad idea in the 5th transition to cancel out the numerator and denominator simply by division. The two O-s in this case may stand for different classes of functions.

Permalink
Share post
Valentin Fadeev

Surface of revolution

Visible to anyone in the world
Edited by Valentin Fadeev, Sunday, 16 Jan 2011, 23:17

This is a simple geometric derivation of the formula for the area of the surface of revolution:

cone

If the surface area of a cone is expressed as:

equation sequence cap s sub c equals integral over zero under two times pi one divided by two times cap l times cap r d phi equals pi times cap r times cap l

Then the area of the frustum is the difference between the whole cone and the cut-away part:

equation sequence cap s sub f equals cap s minus s equals pi times cap r times cap l minus pi times r times open cap l minus l close equals pi times cap r times cap l minus pi times cap r times cap l minus l divided by cap l times open cap l minus l close equals pi times cap r times open cap l minus left parenthesis cap l minus l times right parenthesis squared divided by cap l close equals pi times cap r times l times open one plus cap l minus l divided by cap l close equals pi times open cap r plus r close times l

So, in the infinitesimal case:

equation sequence delta times cap s equals pi times open sum with, 3 , summands y plus y plus delta times y close times delta times l equals two times pi times y times delta times l plus o of delta times y equals two times pi times y times Square root of one plus y super prime two times delta times x plus o of delta times x

Permalink
Share post
Valentin Fadeev

On variable changes in definite integrals

Visible to anyone in the world
Edited by Valentin Fadeev, Sunday, 16 Jan 2011, 23:18

Some integrals yield only one type of substitution that really brings them into a convenient form. Any other method would make them more complicated. However, in some cases totally different methods can be applied with equal effect. In case of definite integrals it is of course not necessary to come back to original variable which makes things even easier. Here is one example

integral over zero under one x cubed times d times x divided by Square root of one minus x squared

The most natural way is to apply a trigonometric substitute. We will not consider this method here. Instead an algebraic trick can be employed:

equation sequence integral over zero under one x cubed times d times x divided by Square root of one minus x squared equals integral over zero under one x squared times x times d times x divided by Square root of one minus x squared equals one divided by two times integral over zero under one x squared times d of x squared divided by Square root of one minus x squared equals one divided by two times integral over zero under one t times d times t divided by Square root of one minus t equals

equation sequence negative one divided by two times integral over zero under one open one minus t minus one close times d times t divided by Square root of one minus t equals one divided by two times integral over zero under one d times t divided by Square root of one minus t minus one divided by two times integral over zero under one Square root of one minus t d t equals

negative Square root of one minus t times vertical line sub zero super one plus one divided by three left parenthesis equation sequence one minus t times right parenthesis super three solidus two times vertical line sub zero super one equals one minus one divided by three equals two divided by three

Alternatively we can use integration by parts:

equation sequence integral over zero under one x cubed times d times x divided by Square root of one minus x squared equals integral over zero under one x squared times x times d times x divided by Square root of one minus x squared equals

equation sequence equals integral over zero under one x squared times d of negative Square root of one minus x squared equals negative x squared times Square root of one minus x squared times vertical line sub zero super one plus integral over zero under one Square root of one minus x squared times two times x d x equals

equation left hand side equals right hand side negative two divided by three left parenthesis equation left hand side one minus x squared times right parenthesis super three solidus two times vertical line sub zero super one equals right hand side two divided by three

Or apply an even more exotic treatment:

let x equals one divided by t

equation left hand side integral over zero under one x cubed times d times x divided by Square root of one minus x squared equals right hand side integral over one under normal infinity d times t divided by t super four times Square root of t squared minus one

let t equals hyperbolic cosine of z

equation sequence integral over one under normal infinity d times t divided by t super four times Square root of t squared minus one equals integral over hyperbolic cosine super negative one of one under normal infinity hyperbolic sine of z times d times z divided by left parenthesis hyperbolic cosine of z times right parenthesis super four times hyperbolic sine of z equals integral over hyperbolic cosine super negative one of one under normal infinity d times z divided by left parenthesis hyperbolic cosine of z times right parenthesis super four equals

equation sequence equals integral over hyperbolic cosine super negative one of one under normal infinity left parenthesis left parenthesis hyperbolic cosine of z times right parenthesis squared minus open hyperbolic sine of z times right parenthesis squared close times d times z divided by left parenthesis hyperbolic cosine of z times right parenthesis super four equals

equation sequence equals integral over hyperbolic cosine super negative one of one under normal infinity d times z divided by left parenthesis hyperbolic cosine of z times right parenthesis squared minus integral over hyperbolic cosine super negative one of one under normal infinity left parenthesis hyperbolic tangent of z times right parenthesis squared times d times z divided by left parenthesis hyperbolic cosine of z times right parenthesis squared equals

MathJax failure: TeX parse error: Extra open brace or missing close brace

For

lim over z right arrow normal infinity of hyperbolic tangent of z equals one

and

equation sequence hyperbolic tangent of hyperbolic cosine super negative one of one equals Square root of one minus one divided by left parenthesis hyperbolic cosine of open hyperbolic cosine super negative one of one close times right parenthesis squared equals zero ,

the same result is obtained

Permalink
Share post
Valentin Fadeev

Summing multiplier

Visible to anyone in the world
Edited by Valentin Fadeev, Sunday, 16 Jan 2011, 23:19

This example illustrates the application of the method of "summing multiplier" to solving certain types in recurrencies. It can be found for instance in the book "Concrete Mathematics" by Graham, Knuth and Patashnik. In its essence it translates the idea of the integrating multiplier from the theory of ODEs.

 

Consider the recurrency:

equation left hand side cap i sub n equals right hand side negative n divided by a times cap i sub n minus one plus one divided by a times x super n times e super a times x

Rewrite it in the form

equation left hand side a times cap i sub n equals right hand side negative n times cap i sub n minus one plus x super n times e super a times x

then multiply both sides by s sub n which is to be determined:

equation left hand side s sub n times a times cap i sub n equals right hand side negative s sub n times n times cap i sub n minus one plus s sub n times x super n times e super a times x

The trick will be done, if we find s sub n satisfying the relation:

equation left hand side negative s sub n times n equals right hand side s sub n minus one times a

Solving for s sub n and expanding recursively we get:

equation left hand side s sub n equals right hand side left parenthesis negative one times right parenthesis super n minus one times a super n divided by n factorial

substituiting into equation:

left parenthesis negative one times right parenthesis super n minus one times a super n plus one divided by n factorial times cap i sub n postfix minus left parenthesis equation left hand side negative one times right parenthesis super n minus two times a super n divided by open n minus one close factorial times cap i sub n minus one equals right hand side left parenthesis negative one times right parenthesis super n minus one times a super n divided by n factorial times x super n times e super a times x

cap i sub zero is easily obtained from the original integral:

equation left hand side cap i sub zero equals right hand side e super a times x minus one divided by a

Summing from 1 to n we get the so called "telescopic sum" on the left side meaning that only the first and the last terms survive:

left parenthesis equation left hand side negative one times right parenthesis super n minus one times a super n plus one divided by n factorial times cap i sub n minus open negative a close times cap i sub zero equals right hand side e super a times x times n ary summation from k equals one to n over left parenthesis negative one times right parenthesis super k minus one times a super k divided by k factorial times x super k

Ultimately solve for cap i sub n :

equation left hand side cap i sub n equals right hand side minus left parenthesis negative one times right parenthesis super n times n factorial divided by a super n plus one left parenthesis equation left hand side one minus e super a times x minus e super a times x times n ary summation from k equals one to n over open negative one times right parenthesis super k times a super k divided by k factorial times x super k close equals right hand side minus left parenthesis negative one times right parenthesis super n times n factorial divided by a super n plus one left parenthesis one minus n ary summation from k equals zero to n over open negative one times right parenthesis super k times a super k divided by k factorial times x super k close equals n factorial e super a times x times n ary summation from k equals zero to n over left parenthesis negative one times right parenthesis super n minus k divided by a super n minus k plus one times k factorial minus left parenthesis negative one times right parenthesis super n times n factorial divided by a super n plus one

(Note that the second term on the right side is the solution for "homogeneous" variant of the equation, i.e. withot x super n times e super a times x term, suggesting another method borrowed from ODEs)

Permalink
Share post
Valentin Fadeev

Generalized homogeneous equations

Visible to anyone in the world
Edited by Valentin Fadeev, Sunday, 16 Jan 2011, 23:20

One of the rarely used methods of solving ODEs applies to the so-called generalized homogeneous equations. The word "generalized" means that the terms are not homogeneous in the classic sense, if all variables are assigned the same dimension. But they may be made homogeneous in a wider sense by choosing the appropriate dimension for the dependent variable. Here is one example.

equation left hand side x cubed times d times y divided by d times x equals right hand side y times open x squared plus y close

If we assign dimension 1 to x and dx and dimension m to y and dy, then the left side has dimension 3+m-1=m+2 on the right side we have m+2 and 2m. To balance things let m+2=2m, hence m=2 and we get a "generalized homogeneous equation" of the 4th order. The trick is to let:

x equals e super t comma y equals u times e super m times t

which in this case gives:

equation left hand side d times x equals right hand side e super t times d times t comma equation left hand side d times y equals right hand side e super two times t times open u super prime plus two times u close

equation left hand side d times y divided by d times x equals right hand side e super t times open u super prime plus two times u close

Hence the equation becomes:

equation left hand side e super four times t times open u super prime plus two times u close equals right hand side u times e super four times t times open one plus u close

equation left hand side u super prime plus u equals right hand side u squared

u super prime divided by u squared plus one divided by u equals one

letting z=1/y

equation left hand side d times z divided by d times x equals right hand side z minus one

z equals one plus c times x

y equals x squared divided by one plus c times x

This method can of course, be applied to higher order equations

Permalink
Share post

This blog might contain posts that are only visible to logged-in users, or where only logged-in users can comment. If you have an account on the system, please log in for full access.

Total visits to this blog: 54002